0 Daumen
475 Aufrufe

Beweisen Sie: $$\frac{x^2y^2+y^2z^2+z^2x^2}{xyz} \geq \sqrt{3}$$, wenn $$x,y,z >0$$ und $$x^2+y^2+z^2=1$$

Avatar von

Hallo,

es gibt eine einfache rechnerische Bearbeitung, wenn man mehrdimensionale Analysis verwenden darf (Minimierung mit Nebenbedingung). Würde Dir das helfen. Oder suchst Du nach einer "elementaren" Herleitung, alleine mit den Rechenregeln für reelle Zahlen?

Gruß

Falls Du eine Lösung hast, würde ich mich freuen, wenn du sie hier hochlädst. (auch durch mehrdimensionale Analysis). Eine einfache Lösung wäre aber natürlich besser.

Entschuldigung: Ich war vorschnell mit meiner Bemerkung zur mehrdim. Analysis, ist tatsächlich nicht einfach.

Gruß

Mit mehrdimensionaler Analysis kann man leicht unter Computereinsatz die Ungleichung bestätigen. Wenn du es elementarer willst, würde ich zwei Dinge vorschlagen:

(1) AM-GM-Ungleichung

(2) Kugelkoordinanten + geometrische Ungleichungen

Manchmal hilft auch Cauchy-Schwarz bei sowas.

Disclaimer: Ich habe das noch nicht ausprobiert, sind aber einige Anregungen.

Meine Antwort ist jetzt vollständig, nicht schön, aber einfach, falls ich mich zwischendurch nicht verrechnet habe, sollte es passen.

1 Antwort

0 Daumen

$$\frac{x^2y^2+y^2z^2+z^2x^2}{xyz} \geq \sqrt{3}$$

$$(x^2y^2+y^2z^2+z^2x^2)^2 \geq 3 x^2y^2z^2$$

$$(x^4y^4+y^4z^4+z^4x^4)+2(x^4y^2z^2+x^2y^4z^2+x^2y^2z^4)\geq 3 x^2y^2z^2$$

$$(x^4y^4+y^4z^4+z^4x^4)+2(x^2+y^2+z^2)x^2y^2z^2\geq 3 x^2y^2z^2$$

$$(x^4y^4+y^4z^4+z^4x^4)+2x^2y^2z^2\geq 3 x^2y^2z^2$$


$$(x^4y^4+y^4z^4+z^4x^4)\geq  x^2y^2z^2$$

$$falls;  x^2=y^2 =z^2=\frac{1}{3}$$

$$3(\frac{1}{3})^2(\frac{1}{3})^2=\frac{1}{27}=\frac{1}{3}*\frac{1}{3}*\frac{1}{3}$$

$$ x^2-a=y^2-b =z^2-c=\frac{1}{3}$$

$$1 \geq a ; b ; c \geq -1$$

$$a+b+c=0$$
$$(\frac{1}{3}+a)^2(\frac{1}{3}+b)^2+(\frac{1}{3}+b)^2(\frac{1}{3}+c)^2+(\frac{1}{3}+c)^2(\frac{1}{3}+a)^2 \geq (\frac{1}{3}+a)*(\frac{1}{3}+b)*(\frac{1}{3}+c)$$

$$(\frac{1}{9}+\frac{2}{3}a+a^2)(\frac{1}{9}+\frac{2}{3}b+b^2)+ (\frac{1}{9}+\frac{2}{3}b+b^2)+(\frac{1}{9}+\frac{2}{3}c+c^2)(\frac{1}{9}+\frac{2}{3}a+a^2)\geq \frac{1}{27}+\frac{1}{9}(a+b+c)+\frac{1}{3}(ab+bc+ca)+abc$$

$$(\frac{1}{81}+\frac{2}{27}(a+b)+\frac{1}{9}(a^2+b^2)+ \frac{4}{9}(ab)+\frac{2}{3}(a^2b+ab^2)+a^2b^2)$$$$+(\frac{1}{81}+\frac{2}{27}(b+c)+\frac{1}{9}(b^2+c^2)+ \frac{4}{9}(bc)+\frac{2}{3}(b^2c+bc^2)+b^2c^2)$$$$+(\frac{1}{81}+\frac{2}{27}(a+c)+\frac{1}{9}(a^2+c^2)+ \frac{4}{9}(ac)+\frac{2}{3}(a^2c+ac^2)+a^2c^2)$$$$ \geq \frac{1}{27}+\frac{1}{9}(a+b+c)+\frac{1}{3}(ab+bc+ca)+abc$$

$$(\frac{1}{9}(a^2+b^2)+ \frac{4}{9}(ab)+\frac{2}{3}(a^2b+ab^2)+a^2b^2)$$$$+(\frac{1}{9}(b^2+c^2)+ \frac{4}{9}(bc)+\frac{2}{3}(b^2c+bc^2)+b^2c^2)$$$$+(\frac{1}{9}(a^2+c^2)+ \frac{4}{9}(ac)+\frac{2}{3}(a^2c+ac^2)+a^2c^2)$$$$ \geq \frac{1}{3}(ab+bc+ca)+abc$$

$$(\frac{2}{3}(a^2+b^2+c^2)+ \frac{4}{9}(ab+bc+ca)+\frac{2}{3}(a^2(b+c)+b^2(c+a)+c^2(a+b))+(a^2b^2+b^2c^2+c^2a^2)$$$$ \geq \frac{1}{3}(ab+bc+ca)+abc$$

$$(\frac{2}{3}(a^2+b^2+c^2)+ \frac{1}{9}(ab+bc+ca)+\frac{2}{3}(a^2(b+c)+b^2(c+a)+c^2(a+b))+(a^2b^2+b^2c^2+c^2a^2)$$$$ \geq abc$$

Eine Idee ist c durch - (a+b) zu ersetzen.

$$1 \geq a \geq b \geq c \geq -1$$

$$c=- (a+b)$$

$$(\frac{4}{3}(a^2+ab+b^2)- \frac{1}{9}(a^2+ab+b^2)+2(a^2b+ab^2)+(a^2b^2+(b^2+a^2)(a+b)^2)$$$$ \geq -a^2b-ab^2$$

$$(\frac{11}{3}(a^2+ab+b^2)+3(a^2b+ab^2)+(a^2b^2+(b^2+a^2)(a+b)^2)\geq 0$$ä

Stimmt für

$$1 \geq a \geq b \geq 0 \geq c \geq -1$$

Wenn

$$1 \geq a \geq 0 \geq b \geq c \geq -1$$

So ist

$$1 |a| \geq |b|$$

$$(\frac{11}{3}(a+b)2-ab)+3(a^2b+ab^2)+(a^2b^2+(b^2+a^2)(a+b)^2)\geq 0$$ä

Wenn b<0, dann

$$\frac{11}{3} (-ab)+3a^2b>0$$

Damit ist die Behauptung bewiesen.

Avatar von 11 k

Ein anderes Problem?

Stell deine Frage

Willkommen bei der Mathelounge! Stell deine Frage einfach und kostenlos

x
Made by a lovely community